K
Khách

Hãy nhập câu hỏi của bạn vào đây, nếu là tài khoản VIP, bạn sẽ được ưu tiên trả lời.

16 tháng 4 2020

Chịu khó ghi ra đề luôn ak em,ghi sigma thế nhiều người khó hiểu :((

Đề là như thế này:Cho a,b,c là các số dương,chứng minh rằng:

\(\frac{a^2+b^2}{a+b}+\frac{b^2+c^2}{b+c}+\frac{c^2+a^2}{c+a}\le\frac{3\left(a^2+b^2+c^2\right)}{a+b+c}\)

\(\Leftrightarrow\left(a+b+c\right)\left(\frac{a^2+b^2}{a+b}+\frac{b^2+c^2}{b+c}+\frac{c^2+a^2}{c+a}\right)\le3\left(a^2+b^2+c^2\right)\)

\(\Leftrightarrow\frac{\left[\left(a+b\right)+c\right]\left(a^2+b^2\right)}{a+b}+\frac{\left[\left(b+c\right)+a\right]\left(b^2+c^2\right)}{b+c}+\frac{\left[\left(c+a\right)+b\right]\left(c^2+a^2\right)}{c+a}\le3\left(a^2+b^2+c^2\right)\)

\(\Leftrightarrow\frac{c\left(a^2+b^2\right)}{a+b}+\frac{a\left(b^2+c^2\right)}{b+c}+\frac{b\left(a^2+c^2\right)}{a+c}\le a^2+b^2+c^2\)

\(\Leftrightarrow c^2-\frac{c\left(a^2+b^2\right)}{a+b}+a^2-\frac{a\left(b^2+c^2\right)}{b+c}+b^2-\frac{b\left(a^2+c^2\right)}{a+c}\ge0\)

\(\Leftrightarrow\frac{ac\left(c-a\right)^2}{\left(a+b\right)\left(b+c\right)}+\frac{bc\left(b-c\right)^2}{\left(a+b\right)\left(a+c\right)}+\frac{ca\left(a-b\right)^2}{\left(a+c\right)\left(b+c\right)}\ge0\) ( đúng )

16 tháng 4 2020

Chuyên Nguyễn Bình Khiêm Quảng Nam 2018

Bá đạo sever là tao cho xin cái đề

15 tháng 9 2017

đề dòng đầu đó :V THêm a,b,c>0 nhé :v

NV
19 tháng 12 2020

\(VT=\sum\dfrac{a^2}{a+3abc+4\left(ab+ac\right)}\ge\dfrac{\left(a+b+c\right)^2}{a+b+c+9abc+8\left(ab+bc+ca\right)}\)

\(VT\ge\dfrac{\left(a+b+c\right)^2}{a+b+c+\dfrac{1}{3}\left(a+b+c\right)^3+\dfrac{8}{3}\left(a+b+c\right)^2}=\dfrac{1}{4}\)

Dấu "=" xảy ra khi và chỉ khi \(a=b=c=\dfrac{1}{3}\)

Đề bài bị sai con số bên vế phải

27 tháng 7 2019

Haiz giải ra rồi

Ta có : \(VT=\Sigma\left(\frac{a^2-bc}{2ka^2+k^2b^2+c^2}\right)\ge0\)

\(\Leftrightarrow\Sigma\left(\frac{2k\left(a^2-bc\right)}{2ka^2+k^2b^2+c^2}\right)\ge0\)

\(\Leftrightarrow\Sigma\left(\frac{2ka^2-2kbc}{2ka^2+k^2b^2+c^2}\right)\ge0\)

\(\Leftrightarrow\Sigma\left(\frac{2ka^2+k^2b^2+c^2+2ka^2-2kbc-2ka^2-k^2b^2-c^2}{2ka^2+k^2b^2+c^2}\right)\ge0\)

\(\Leftrightarrow\Sigma\left(1-\frac{2kbc-2ka^2+2ka^2+k^2b^2+c^2}{2ka^2+k^2b^2+c^2}\right)\ge0\)

\(\Leftrightarrow\Sigma\left(1-\frac{k^2b^2+2kbc+c^2}{\left(k^2b^2+ka^2\right)+\left(ka^2+c^2\right)}\right)\ge0\)

\(\Leftrightarrow\Sigma\left(1-\frac{\left(kb+c\right)^2}{\left(k^2b^2+ka^2\right)+\left(ka^2+c^2\right)}\right)\ge0\)

Áp dụng bất đẳng thức Cauchy-Schwarz :

\(VT=\Sigma\left(1-\frac{\left(kb+c\right)^2}{\left(k^2b^2+ka^2\right)+\left(ka^2+c^2\right)}\right)\ge\Sigma\left[1-\left(\frac{k^2b^2}{k^2b^2+ka^2}+\frac{c^2}{ka^2+c^2}\right)\right]\)

\(=3-\left(\frac{k^2b^2+ka^2}{k^2b^2+ka^2}+\frac{ka^2+c^2}{ka^2+c^2}+\frac{k^2b^2+c^2}{k^2b^2+c^2}\right)=3-3=0\)( đpcm )

Dấu "=" xảy ra \(\Leftrightarrow\left\{{}\begin{matrix}a=b=c\\k>0\end{matrix}\right.\)

27 tháng 7 2019

Ta có: \(1-\frac{2k\left(a^2-bc\right)}{2ka^2+k^2b^2+c^2}=\frac{\left(kb+c\right)^2}{2ka^2+k^2b^2+c^2}\)

Ta có thể viết lại bất đẳng thức thành

\(\sum\frac{\left(kb+c\right)^2}{2ka^2+k^2b^2+c^2}\le3\)

Sử dụng BĐT Cauchy-Schwarz, ta có:

\(\frac{\left(kb+c\right)^2}{2ka^2+k^2b^2+c^2}=\frac{\left(kb+c\right)^2}{k\left(a^2+kb^2\right)+c^2+ka^2}\le\frac{kb^2}{a^2+kb^2}+\frac{c^2}{c^2+kc^2}\)

Tương tự rồi cộng lại, ta có điều phải chứng minh. Đẳng thức xảy ra khi \(a=b=c\), hoặc \(a=\frac{b}{k}=\frac{c}{k^2}\), hoặc \(b=\frac{c}{k}=\frac{a}{k^2}\), hoặc \(c=\frac{a}{k}=\frac{b}{k^{^2}}\)

Hoặc ta có thể làm như sau.

\(\frac{\left(kb+c\right)^2}{2ka^2+k^2b^2+c^2}=\frac{kb^2}{a^2+kb^2}+\frac{c^2}{c^2+kc^2}-\frac{k\left(a^2-bc\right)^2\left(kb-c\right)^2}{\left(a^2+kb^2\right)\left(c^2+kc^2\right)\left(2ka^2+k^2b^2+c^2\right)}\)

Ta có đẳng thức sau:

\(\sum\frac{\left(kb+c\right)^2}{2ka^2+k^2b^2+c^2}=3-p\sum\frac{\left(a^2-bc\right)^2\left(kb-c\right)^2}{\left(a^2+kb^2\right)\left(c^2+ka^2\right)\left(2ka^2+k^2b^2+c^2\right)}\)

\(\sum\frac{a^2-bc}{2ka^2+k^2b^2+c^2}=\frac{1}{2}\sum\frac{\left(a^2-bc\right)^2\left(kb-c\right)^2}{\left(a^2+kb^2\right)\left(c^2+ka^2\right)\left(2ka^2+k^2b^2+c^2\right)}\)

Do đó, bất đẳng thức ban đầu tương đương với

\(\sum\frac{\left(b^2+kc^2\right)\left(a^2-bc\right)^2\left(kb-c\right)^2}{2ka^2+k^2b^2+c^2}\ge0\)

1 tháng 1 2021

giả sử \(a\ge b\ge c\ge0\)

Ta có: \(a+\frac{b}{2}-\frac{a^2+ab+b^2}{a+b}=\frac{1}{2}\left(ab-b^2\right)\ge0\Rightarrow a+\frac{b}{2}\ge\frac{a^2+ab+b^2}{a+b}\)

\(b+\frac{a}{2}-\frac{a^2+ab+b^2}{a+b}=\frac{1}{2}\left(ab-a^2\right)\le0\Rightarrow b+\frac{a}{2}\le\frac{a^2+ab+b^2}{a+b}\)

Tương tự: \(b+\frac{c}{2}\ge\frac{b^2+bc+c^2}{b+c}\ge c+\frac{b}{2};a+\frac{c}{2}\ge\frac{a^2+ac+c^2}{a+c}\ge c+\frac{a}{2}\)

Lại có:+) \(\frac{a^3-b^3}{a+b}+\frac{b^3-c^3}{b+c}+\frac{c^3-a^3}{c+a}\)

\(=\left(a-b\right)\frac{a^2+ab+b^2}{a+b}+\left(b-c\right)\frac{b^2+bc+c^2}{b+c}-\left(a-c\right)\frac{a^2+ac+c^2}{a+c}\)

\(\ge\left(a-b\right)\left(b+\frac{a}{2}\right)+\left(b-c\right)\left(c+\frac{a}{2}\right)-\left(a-c\right)\left(a+\frac{c}{2}\right)\)

\(\ge\frac{-1}{4}\left[\left(a-b\right)^2+\left(b-c\right)^2+\left(c-a\right)^2\right]\left(1\right)\)

+) \(\frac{a^3-b^3}{a+b}+\frac{b^3-c^3}{b+c}+\frac{c^3-a^3}{c+a}\)

\(=\left(a-b\right)\frac{a^2+ab+b^2}{a+b}+\left(b-c\right)\frac{b^2+bc+c^2}{b+c}-\left(a-c\right)\frac{a^2+ac+c^2}{a+c}\)

\(\le\left(a-b\right)\left(a+\frac{b}{2}\right)+\left(b-c\right)\left(b+\frac{c}{2}\right)-\left(a-c\right)\left(c+\frac{a}{2}\right)\)

\(\le\frac{1}{4}\left[\left(a-b\right)^2+\left(b-c\right)^2+\left(c-a\right)^2\right]\left(2\right)\)

Từ 1,2 => đpcm

2 tháng 1 2021

BĐT đã cho tuong duong voi:

\(\left|\frac{\left(a-b\right)\left(b-c\right)\left(c-a\right)\left(ab+bc+ca\right)}{\left(a+b\right)\left(b+c\right)\left(c+a\right)}\right|\le\frac{1}{4}\left[\Sigma\left(a-b\right)^2\right]\)

Theo AM-GM ta có: \(\left(ab+bc+ca\right)\le\frac{9}{8}\cdot\frac{\left(a+b\right)\left(b+c\right)\left(c+a\right)}{a+b+c}\)

Có: \(VT\le\frac{9}{8}\left|\frac{\sqrt{\left(a-b\right)^2\left(b-c\right)^2\left(c-a\right)^2}}{\left(a+b+c\right)}\right|=\frac{9\sqrt{\left(a-b\right)^2\left(b-c\right)^2\left(c-a\right)^2}}{8\left(a+b+c\right)}\)

Cần chứng minh: \(4\left(a+b+c\right)^2\left[\Sigma\left(a-b\right)^2\right]^2\ge9\left(a-b\right)^2\left(b-c\right)^2\left(c-a\right)^2\)

Rõ ràng \(\Sigma\left(a-b\right)^2\ge3\sqrt[3]{\left(a-b\right)^2\left(b-c\right)^2\left(c-a\right)^2}\)

Cần cm: \(36\left(a+b+c\right)^2\sqrt[3]{\left(a-b\right)^4\left(b-c\right)^4\left(c-a\right)^4}\ge9\sqrt[3]{\left(a-b\right)^6\left(b-c\right)^6\left(c-a\right)^6}\)

Hay \(4\left(a+b+c\right)^2\ge\sqrt[3]{\left(a-b\right)^2\left(b-c\right)^2\left(c-a\right)^2}\)

Tiếp tục là điều hiển nhiên do \(VT\ge4\left[\left(a+b+c\right)^2-3\left(ab+bc+ca\right)\right]\)

\(=2\left[\left(a-b\right)^2+\left(b-c\right)^2+\left(c-a\right)^2\right]\)

\(\ge6\sqrt[3]{\left(a-b\right)^2\left(b-c\right)^2\left(c-a\right)^2}\ge VP\)

Đẳng thức xảy ra khi \(\hept{\begin{cases}\left(a-b\right)\left(b-c\right)\left(c-a\right)=0\\a-b=b-c=c-a\\a=b=c\end{cases}}\Leftrightarrow a=b=c.\)

7 tháng 5 2020

\(\Leftrightarrow\Sigma\sqrt{\frac{3a^3}{\left[5a^2+\left(b+c\right)^2\right]\left(a+b+c\right)}}\le1\)

Theo Am-GM: \(VT=\Sigma\sqrt{\frac{3a^2}{5a^2+\left(b+c\right)^2}.\frac{a}{a+b+c}}\le\Sigma\frac{3a^2}{2\left(5a^2+\left(b+c\right)^2\right)}+\frac{1}{2}\)

Như vậy nó là đủ để chứng minh rằng: \(\Sigma\frac{3a^2}{5a^2+\left(b+c\right)^2}\le1\)

Giả sử \(c=min\left\{a,b,c\right\}\) nó tương đương:

$$2\, \left( a-b \right) ^{2} \left( 3\,c+a+b \right) \left( -c+a+b
 \right) \left( {a}^{2}+2\,ab+{b}^{2}+5\,{c}^{2} \right) +2\,c
 \left( a-c \right) \left( b-c \right) \left( 3\,{a}^{3}+9\,{a}^{2}b
+17\,c{a}^{2}+9\,a{b}^{2}-20\,abc+3\,{c}^{2}a+3\,{b}^{3}+17\,c{b}^{2}+
3\,{c}^{2}b+{c}^{3} \right) \geqq 0$$

(Gõ Latex, không hiện thì vô thống kê hỏi đáp xem)

Đây là điều hiển nhiên/

PS: Bài này quan trọng là ý tưởng phá căn thôi chứ không có gì khó. Lúc đầu UCT bất đẳng thức cuối cho đẹp nhưng phải xét các TH mệt lắm, chưa rành nên không làm cách đó:D

7 tháng 5 2020

Chứng minh: \(\Sigma\frac{3a^2}{5a^2+\left(b+c\right)^2}\le1\), cách 2:

Đổi biến sang pqr: (Vô thống kê hỏi đáp xem nếu olm không hiện Latex)

Nếu \(p^2\le4q\) ta cần:

$$2/9\,p \left( 19\,{p}^{2}-36\,q \right) \left( {p}^{3}-4\,qp+9\,r
 \right) -4/9\, \left( {p}^{2}-3\,q \right) \left( {p}^{2}-4\,q
 \right) \left( 5\,{p}^{2}-3\,q \right) \geqq 0$$

(Hiển nhiên)

Nếu \(p^2\ge4q\) thì cần chứng minh:

$$2\,p \left( 19\,{p}^{2}-36\,q \right) r+2\, \left( {p}^{2}-4\,q
 \right) \left( {p}^{4}-2\,{q}^{2} \right) \geqq 0$$

(Hiển nhiên)

Từ 2 TH trên ta thu được điều phải chứng minh.

2 tháng 12 2020

Đặt \(\left(\frac{1}{a},\frac{1}{b},\frac{1}{c}\right)=\left(x,y,z\right)\)

\(x+y+z\ge\frac{x^2+2xy}{2x+y}+\frac{y^2+2yz}{2y+z}+\frac{z^2+2zx}{2z+x}\)

\(\Leftrightarrow x+y+z\ge\frac{3xy}{2x+y}+\frac{3yz}{2y+z}+\frac{3zx}{2z+x}\)

\(\frac{3xy}{2x+y}\le\frac{3}{9}xy\left(\frac{1}{x}+\frac{1}{x}+\frac{1}{y}\right)=\frac{1}{3}\left(x+2y\right)\)

\(\Rightarrow\Sigma_{cyc}\frac{3xy}{2x+y}\le\frac{1}{3}\left[\left(x+2y\right)+\left(y+2z\right)+\left(z+2x\right)\right]=x+y+z\)

Dấu "=" xảy ra khi x=y=z

28 tháng 8 2020

Áp dụng giả thiết và một đánh giá quen thuộc, ta được: \(16\left(a+b+c\right)\ge\frac{1}{a}+\frac{1}{b}+\frac{1}{c}=\frac{ab+bc+ca}{abc}=\frac{\left(ab+bc+ca\right)^2}{abc\left(ab+bc+ca\right)}\ge\frac{3\left(a+b+c\right)}{ab+bc+ca}\)hay \(\frac{1}{6\left(ab+bc+ca\right)}\le\frac{8}{9}\)

Đến đây, ta cần chứng minh \(\frac{1}{\left(a+b+\sqrt{2\left(a+c\right)}\right)^3}+\frac{1}{\left(b+c+\sqrt{2\left(b+a\right)}\right)^3}+\frac{1}{\left(c+a+\sqrt{2\left(c+b\right)}\right)^3}\le\frac{1}{6\left(ab+bc+ca\right)}\)

 Áp dụng bất đẳng thức Cauchy cho ba số dương ta có \(a+b+\sqrt{2\left(a+c\right)}=a+b+\sqrt{\frac{a+c}{2}}+\sqrt{\frac{a+c}{2}}\ge3\sqrt[3]{\frac{\left(a+b\right)\left(a+c\right)}{2}}\)hay \(\left(a+b+\sqrt{2\left(a+c\right)}\right)^3\ge\frac{27\left(a+b\right)\left(a+c\right)}{2}\Leftrightarrow\frac{1}{\left(a+b+2\sqrt{a+c}\right)^3}\le\frac{2}{27\left(a+b\right)\left(a+c\right)}\)

Hoàn toàn tương tự ta có \(\frac{1}{\left(b+c+2\sqrt{b+a}\right)^3}\le\frac{2}{27\left(b+c\right)\left(b+a\right)}\)\(\frac{1}{\left(c+a+2\sqrt{c+b}\right)^3}\le\frac{2}{27\left(c+a\right)\left(c+b\right)}\)

Cộng theo vế các bất đẳng thức trên ta được \(\frac{1}{\left(a+b+\sqrt{2\left(a+c\right)}\right)^3}+\frac{1}{\left(b+c+\sqrt{2\left(b+a\right)}\right)^3}+\frac{1}{\left(c+a+\sqrt{2\left(c+b\right)}\right)^3}\le\frac{4\left(a+b+c\right)}{27\left(a+b\right)\left(b+c\right)\left(c+a\right)}\)Phép chứng minh sẽ hoàn tất nếu ta chỉ ra được \(\frac{4\left(a+b+c\right)}{27\left(a+b\right)\left(b+c\right)\left(c+a\right)}\le\frac{1}{6\left(ab+bc+ca\right)}\)\(\Leftrightarrow\left(a+b\right)\left(b+c\right)\left(c+a\right)\ge\frac{8}{9}\left(ab+bc+ca\right)\left(a+b+c\right)\)

Đây là một đánh giá đúng, thật vậy: đặt a + b + c = p; ab + bc + ca = q; abc = r thì bất đẳng thức trên trở thành \(pq-r\ge\frac{8}{9}pq\Leftrightarrow\frac{1}{9}pq\ge r\)*đúng vì \(a+b+c\ge3\sqrt[3]{abc}\)\(ab+bc+ca\ge3\sqrt[3]{\left(abc\right)^2}\))

Vậy bất đẳng thức được chứng minh

Đẳng thức xảy ra khi \(a=b=c=\frac{1}{4}\)